LSAT and Law School Admissions Forum

Get expert LSAT preparation and law school admissions advice from PowerScore Test Preparation.

 Administrator
PowerScore Staff
  • PowerScore Staff
  • Posts: 8916
  • Joined: Feb 02, 2011
|
#24977
Complete Question Explanation

Must Be True—SN. The correct answer choice is (D)

As is common in Must Be True questions, this stimulus contains conditional reasoning. The author sets out the rules for members of the Frequent Viewers club to obtain a special coupon. If the member has rented more than ten movies in the last month, they can only obtain the coupon at the store from which they last rented. If they rented ten or fewer, they can only obtain the coupon from the Main Street location. From these facts, we can derive two separate conditional rules:
  • Sufficient ..... ..... ..... ..... Necessary

    More than ten ..... :arrow: ..... Coupon at store from which they last rented

    Ten or fewer ..... :arrow: ..... Coupon only from Main Street
The stimulus then provides the information that Pat, who has rented fewer than ten movies, can obtain her coupon at Walnut Lane. Immediately, we can see a problem. The conditional rules above state that if Pat rented ten or fewer movies, she can only obtain the coupon the Main Street store. This apparent conflict is the key to the question, and it is worth the time to figure out what additional information is present in the stimulus. On a closer read, we discover that the conditional rules only apply to those in the Frequent Viewers club. If Pat is not in Frequent Viewers club, the conditional rules above would not apply.

Answer choice (A): This is a mistaken reversal. The conditional rule states that the if the member rents ten or fewer movies, they must get the coupon from the Main Street store, but it does not say the only people who can get the coupon there are members of the Frequent Viewers club with ten or fewer rentals.

Answer choice (B): Even though the stimulus sets out rules for how members of the Frequent Viewers club who rent ten or fewer movies can get the coupon, it does not say that there are any members of the Frequent Viewers club to whom the rule would apply. We do not know anything about the members of the club. We only know the rules of the club, and information about Pat.

Answer choice (C): This answer choice must be false. Either members of the club will have rented ten or fewer videos, in which case they must get the coupon from Main Street, or they will have rented more than ten videos, in which case they must get the coupon from the store from which they last rented. The conditional rules mean that members of the club can only get the coupon from a single specific location.

Answer choice (D): This is the correct answer choice. As discussed above, Pat’s situation did not fit into either conditional rule. The only way that she could have rented ten or fewer movies, and received her location at a store other than Main Street, is if she was not a member of the Frequent Viewers club. This must mean that non-members can also receive coupons.

Answer choice (E): The rules do not tell us anything about when people cannot get coupons; it only gives information about when members can get coupons. As with any Must Be True questions, any answer choice that lacks support in the stimulus is incorrect.
 Applesaid
  • Posts: 29
  • Joined: Oct 18, 2013
|
#12486
Hello!

I am having troubles to deduce that D is the correct answer while C is not.

Since this is a must be true question. There are three major factual statements in the passage.

1) Members of the club can now receive a special discount coupon.

2) Members who have rented more than 10 videos in the past month can receive the coupon only at the location where the member last rented a movie.

3) Members who have not rented more than 10 videos in the past month can receive the coupon only at the MS station.

Pat, who has not rented more than ten videos in the past month, can receive the special discount coupon at the WL location.

If Pat is a member who has not rented more than 10 videos, he must have to receive the coupon at MS station.

Wonder if it's becuz "if Pat does not receive a coupon from MS but WL, then he must not be a member of the club?"

But why not C?
User avatar
 KelseyWoods
PowerScore Staff
  • PowerScore Staff
  • Posts: 1079
  • Joined: Jun 26, 2013
|
#12500
Hi Applesaid!

You are correct that those are the three major factual statements in the stimulus. Statements 2 and 3 are very strong conditional statements (note the "only" in each of them) and together they basically say that whether Frequent Viewers club members have rented more than ten videos in the past month or not, there is ONLY one location they can get the special discount coupon from. Therefore, answer choice (C) doesn't follow because it says that some members can get the coupon from more than one location.

The stimulus also tells us that Pat has not rented more than ten movies in the last month. If Pat is also a member of the Frequent Viewers club, that would mean he would ONLY be able to get the special discount coupon from the Main Street location. But the stimulus tells us that he can get the coupon from the Walnut Lane location. At first, it sounds like Pat is violating our rules! Until we realize that maybe Pat is not a member of the Frequent Viewers club. Our rules only apply to members of the club; we don't have any rules about people who are not members.

Since Pat's example violates the Frequent Viewers club rules, that tells us he isn't a member of the Frequent Viewers club. And since we know that Pat can still receive a special discount coupon, that tells us that some (at least one) non-members can receive the special discount coupon. So answer choice (D) is supported.

Hope that helps!

Best,
Kelsey
 Frank
  • Posts: 16
  • Joined: Apr 30, 2014
|
#14656
Hey,

Couple questions for the first LR section of the October 2011 exam.

#20 - I was really lost for this one. My pre phrase was "The Walnut Lane location is on Main Street" and there was no answer that was even close to that. I ended up settling on B because I though that Pat would've been a member of the club and he didn't rent more than 10 videos (I made a wrong assumption here right?). Do you have to assume that Pat is not a member of the club because they don't say anything about club membership being necessary in order to get the coupon, and they also don't say that Pat is a member?


Thanks for your help.

Frank
User avatar
 KelseyWoods
PowerScore Staff
  • PowerScore Staff
  • Posts: 1079
  • Joined: Jun 26, 2013
|
#14661
Hi Frank,

This is certainly a tricky question! First, I'm going to refer you to an explanation I gave of it in response to another poster's question higher up in this thread.

Here are a couple of additional points specific to your question:

Be careful when prephrasing Must Be True questions. Sometimes, we think we have a good, specific prephrase. But there are often multiple things that Must Be True. So don't get too attached to one specific prephrase. Also, in this case, The Walnut Lane location can't be on Main Street....it's on Walnut Lane! When you don't see your prephrase in the answer choices, try not to get discouraged. Go through each of the options and try to decide whether they are things that absolutely Must Be True.

Answer Choice B could be true, but we don't have enough information to know whether or not some members of the club have not rented more than ten videos. We know that Pat has not rented more than ten videos, but based on the facts we have, we have to conclude that Pat is also not a member of the club since he is not bound by the rules that apply to club members.

I hope this helps!

Best,
Kelsey
 actionjackson
  • Posts: 22
  • Joined: Nov 22, 2016
|
#31371
I'm a little confused by this question. I diagrammed the stimulus like so:

MS= Main Street
LLR=Location Last Rented
~>10 :arrow: MS
>10 :arrow: LLR

I improperly chose C for this question as those rules seem to imply to me that "some members can receive the discount coupon at more than one location" as being consistent with either the location last rented (can be anywhere) or the main street location. Both viable options for a member of the viewers club?
 Adam Tyson
PowerScore Staff
  • PowerScore Staff
  • Posts: 5153
  • Joined: Apr 14, 2011
|
#31379
I don't think there's any support for that answer, action. Imagine you are a member of the club. Now, how many videos have you rented? If it's 10 or fewer, you have one option - you must go to Main Street. You cannot go to more than one location. What if you rented more than 10? Then you must go to the last place you rented. You have no other option. Those are the only choices for members, and every member must, logically and mathematically, be in only one of those categories. There's no way you can simultaneously be in both groups. So no matter what, if you are a member you have only one location at which you can get your coupon. Answer C is a "Cannot Be True" answer - it's impossible!

I hope that helps clear things up! Good luck in your continued studies!
 actionjackson
  • Posts: 22
  • Joined: Nov 22, 2016
|
#31399
Adam Tyson wrote:I don't think there's any support for that answer, action. Imagine you are a member of the club. Now, how many videos have you rented? If it's 10 or fewer, you have one option - you must go to Main Street. You cannot go to more than one location. What if you rented more than 10? Then you must go to the last place you rented. You have no other option. Those are the only choices for members, and every member must, logically and mathematically, be in only one of those categories. There's no way you can simultaneously be in both groups. So no matter what, if you are a member you have only one location at which you can get your coupon. Answer C is a "Cannot Be True" answer - it's impossible!

I hope that helps clear things up! Good luck in your continued studies!
Thanks for the response Mr. Tyson. I'm just wondering, why is it just ten or fewer, and not ten or more as well? Maybe that's where my confusion lies. The stimulus says "more than ten" and "not more than ten". If it were the case that it were ten or more and ten or fewer, then couldn't someone with exactly ten rentals go to any one of two (or endless options I don't know how many fictional Videokings exist)? There's an overlap there or no?
As I'm relooking at this question I'm starting to realize that if Pat were a member, she activated the not less than ten condition and she must go to Main, but how do we get to D? I eliminated that answer immediately thinking we weren't told anything/know anything about non members, and we don't know Pat's membership status.
 Adam Tyson
PowerScore Staff
  • PowerScore Staff
  • Posts: 5153
  • Joined: Apr 14, 2011
|
#31403
"Not more than 10" includes 10, action! If you rented exactly 10, then you did not rent more than 10, and so if you are a member you must get the coupon at Main Street. "Not more than 10" and "more than 10" combine to cover every numeric possibility from 0 to infinity. Careful there, that's where they want to trick you!

(Call me Adam, please. Only my wife calls me Mr. Tyson.)
 salmach
  • Posts: 12
  • Joined: Aug 11, 2017
|
#39165
Hi,

Could you help me figure out if my diagramming for this specific question is off?

Member @ VK :arrow: Coupon (Did I mistake this for ONLY members of the club can receive coupons)? Could you help me understand the difference between what I did and what the stimulus is asking for? Thanks.

10 Videos Past Month :arrow: C @ LL (last location)

If you didn't C@LL, it means you did not order 10 videos past month, which means you can get coupon @ MSL.

So D is correct ... because maybe Walnut Street as different rules than the MSL?

Get the most out of your LSAT Prep Plus subscription.

Analyze and track your performance with our Testing and Analytics Package.